Physics, asked by manojrajoria19pb2h2m, 9 months ago

IN THIS CASE ,
HOW IS R2 AND R3 IN PARALLEL AND R1 IN SERIES​

Attachments:

Answers

Answered by Prabhnoorsinghrehal
0

Answer:

reconstruct the circuit diagram it will become a wheatstone bridge and it would apply condition P/Q=R/S then the resistance on middle wire will be removed.

Thanks

May it will help you

Similar questions